LSAT and Law School Admissions Forum

Get expert LSAT preparation and law school admissions advice from PowerScore Test Preparation.

User avatar
 Dave Killoran
PowerScore Staff
  • PowerScore Staff
  • Posts: 5852
  • Joined: Mar 25, 2011
|
#27295
Complete Question Explanation
(The complete setup for this game can be found here: lsat/viewtopic.php?t=11501)

The correct answer choice is (C)

J must be assigned to a locker with N or T. Thus, J cannot be assigned to a locker with R or F. Because R and F must be assigned separate lockers, that means that two lockers must be reserved for R and F, leaving three lockers that J could be assigned. Hence, answer choice (C) is correct.
 Blueballoon5%
  • Posts: 156
  • Joined: Jul 13, 2015
|
#46577
I am a little confused with this question. I thought that J can be assigned to four different places (1, 2, 4, and 5). It is possible for J to be in these four (I mapped out the choices). It is only not possible for 3 because F is in that place and is a male too.
 Adam Tyson
PowerScore Staff
  • PowerScore Staff
  • Posts: 5153
  • Joined: Apr 14, 2011
|
#47125
That's globally true, Blueballoon, but this question asks about J's options once R has been assigned to a locker. Since R cannot be assigned to a shared locker, once she is assigned then J cannot be assigned to that same locker. Since he also can never be assigned to locker 3, that will leave 3 options, not 4, for J.

For example, if R is assigned to locker 1, then J could only be assigned to locker 2, 4, or 5.

Be sure to include the local restriction information from the question in your analysis!
 iberkley
  • Posts: 6
  • Joined: Jul 04, 2018
|
#49081
Hi!

I have a question about this one- I thought that the correct answer was B, since if F is in 3 and has to be assigned either N or T, and J has to be assigned either N or T, and N&T can't be adjacent, that would mean that J (along with N/T) wouldn't be able to go in either slots 2 or 4. Can someone tell me where I went wrong here?

Thank you!
User avatar
 Jonathan Evans
PowerScore Staff
  • PowerScore Staff
  • Posts: 726
  • Joined: Jun 09, 2016
|
#49150
Hi, IBerkley,

Good question!

F is in 3, but F could be in 3 by himself. J does have to be assigned with N or T, but we are not concerned about J yet on this question.

According to the rule on this local question, we are beginning our setup by assigning R somewhere. As you have observed, F is already assigned. This means that slot 3 is occupied by F (and J cannot go there because we can't have two boys). Also, R is assigned somewhere and cannot share the locker with J because R is alone. Here is one possible scenario:

Image

In this hypothetical, there are three possible options for J: 2, 4, and 5.

I hope this helps!
 iberkley
  • Posts: 6
  • Joined: Jul 04, 2018
|
#49223
that helps a lot, thanks!

Get the most out of your LSAT Prep Plus subscription.

Analyze and track your performance with our Testing and Analytics Package.